Двухуровневая система с синусоидальным управлением (TLS)

Я пытаюсь решить вопрос управляемой двухуровневой системы (TLS или кубит), используя преобразование Фурье уравнения Шредингера (SHE), но я застреваю в решении уравнения.

Данный гамильтониан

ЧАС "=" ( ю 0 2 1 2 В е я т ю Д 1 2 В е я т ю Д ю 0 2 )

и подключаемся к SHE:

я г дт ( с а ( т ) с б ( т ) ) "=" ЧАС ( с а ( т ) с б ( т ) )

Я получаю набор из двух связанных дифференциальных уравнений 1-го порядка, которые я затем преобразую Фурье и использую правило производной и правило сдвига, чтобы получить:

\левый(

ю с а ( ю ) "=" 1 2 В с б ( ю ю Д ) 1 2 ю 0 с а ( ю ) ю с б ( ю ) "=" 1 2 В с а ( ю + ю Д ) + 1 2 ю 0 с б ( ю )
\верно)

Если я сдвину с б уравнение:

ю с б ( ю ю Д ) "=" 1 2 В с а ( ю Д ю Д + ю ) + + 1 2 ю 0 с б ( ю ю Д )

и найдите термин, который я могу снова подключить к первому уравнению:

с б ( ю ю Д ) "=" В с а ( ю ) 2 ю + ю 0

и решить для с а :

с а ( ю ) "=" В 2 с а ( ю ) ( 2 ю ) 2 + ю 0 2

Таким образом, это выглядит как функция Лоренца с шириной резонансной частоты и с центром в 0 умножает с а , но с а отменяет, если это не 0? или какая-то дельта-функция?

Вот тут я не понимаю, как приступить к решению проблемы дальше? Является ли частью проблемы то, что я решаю задачу без граничных условий, просто пытаясь найти устойчивое состояние?

Вы можете найти мой ответ на этот вопрос ( physics.stackexchange.com/questions/138765/… ) полезным.

Ответы (1)

Решение состоит в том, чтобы понять, что стационарное решение системы с гармоническим приводом также должно колебаться гармонически. (Что касается вашего решения, это означает, что спектрально с я ( ю ) являются дельта-функциями, что разрешает ваше противоречие.)

Таким образом, обычно начинают с постулирования колебательного анзаца.

с а ( т ) "=" с а е я ю а т , с б ( т ) "=" с б е я ю б т ,
где с а и с б теперь константы. В качестве анзаца это безвредно, и если окажется, что это не решение, вы можете отказаться от него (но так оно и будет). Таким образом, ваше уравнение Шрёдингера гласит:
( ю 0 2 1 2 В е я т ю Д 1 2 В е я т ю Д ю 0 2 ) ( с а ( т ) с б ( т ) ) "=" я г г т ( с а ( т ) с б ( т ) )
так
( ю 0 2 1 2 В е я т ю Д 1 2 В е я т ю Д ю 0 2 ) ( с а е я ю а т с б е я ю б т ) "=" ( ю а с а е я ю а т ю б с б е я ю б т )
или
{ ю 0 2 с а е я ю а т + 1 2 В е я т ю Д с б е я ю б т "=" ю а с а е я ю а т , 1 2 В е я т ю Д с а е я ю а т + ю 0 2 с б е я ю б т "=" ю б с б е я ю б т .
Это нужно, чтобы вы установили ю а + ю Д "=" ю б , после чего можно устранить зависимость от времени. Это оставляет вас с простой линейной системой
{ ю 0 2 с а + 1 2 В с б "=" ю а с а , + 1 2 В с а + ю 0 2 с б "=" ( ю а + ю Д ) с б ,
которая является системой собственных значений для гамильтониана ЧАС "=" ( ю 0 2 1 2 В 1 2 В ю 0 2 ю Д ) .

Поскольку вы пометили это как , я оставлю расчет здесь, так как уверен, что вам лучше вычислить собственные векторы и собственные значения самостоятельно.

Извините, почему ю а + ю Д "=" ю б ? и какая дельта-функция является решением моего уравнения выше, это дельта ( ю ) или дельта ( ю ю Д ) или дельта ( ю ю 0 ) ?
@AimForClarity Вы хотите, чтобы последнее уравнение с экспонентами сохранялось все время, а это означает, что все экспоненты должны быть идентичными, поэтому их частоты должны совпадать.
Я бы не советовал вам работать с этим представлением Фурье (оно только затемняет тот факт, что компоненты представляют собой просто колеблющиеся экспоненты). Если вы должны это знать, то решите эту последнюю собственную систему для ю а и ю б , и преобразование Фурье исходного анзаца для с а ( т ) и с б ( т ) .
Теперь, когда я думаю об этом, я могу отменить с а в с а ( ю ) "=" В 2 с а ( ю ) ( 2 ю ) 2 + ю 0 2 и найдите омегу, что даст мне точно правильную частоту раби: Ом "=" ( В 2 ) 2 ( ю 0 2 ) 2 . Это говорит мне, что уравнение будет выполняться для любого отличного от 0 или не бесконечного с а и с б только в ю "=" ± Ом , и везде с а и с б должно быть равно 0 (или бесконечности, но это мы не учитываем как нефизическое).
Другими словами, решение представляет собой дельта-функцию при ю "=" или ± Ом с а "=" С а дельта ( ю Ом )  +  С а дельта ( ю Ом ) , где С а , С б любые C-числа.
@AimForClarity, ваш последний комментарий был полностью искажен.
Извините, я все еще печатал, теперь все должно быть там.
Да, в пространстве Фурье это дельта-функция. Но опять же, ничего не получится, если не работать во временной области.